OPTADS360
AANETWORK
AMBIENT
YOMEDIA
Banner-Video
IN_IMAGE

Chứng minh rằng 3

Cho a,b,c\(\in\left[0;2\right]\) và a + b + c = 3. CMR :

\(3\le a^2+b^2+c^2\le5\)

  bởi Phan Thiện Hải 14/01/2019
ADSENSE/lession_isads=0
QUẢNG CÁO
 

Câu trả lời (1)

  • *)Chứng minh \(a^2+b^2+c^2\ge3\)

    Áp dụng BĐT Cauchy-Schwarz ta có:

    \(\left(1+1+1\right)\left(a^2+b^2+c^2\right)\ge\left(a+b+c\right)^2\)

    \(\Leftrightarrow3\left(a^2+b^2+c^2\right)\ge9\Leftrightarrow a^2+b^2+c^2\ge3\)

    Xảy ra khi \(a=b=c=1\)

    *)Chứng minh \(a^2+b^2+c^2\le5\)

    Từ \(a,b,c\in\left[0;2\right]\)\(\Rightarrow\left(2-a\right)\left(2-b\right)\left(2-c\right)\le0\)

    \(\Leftrightarrow2\left(ab+bc+ca\right)-4\left(a+b+c\right)-abc+8\le0\)

    \(\Leftrightarrow2\left(ab+bc+ca\right)-abc\le4\)\(\Leftrightarrow2\left(ab+bc+ca\right)\le4\)

    \(\Leftrightarrow\left(a+b+c\right)^2-a^2+b^2+c^2\le4\)

    \(\Leftrightarrow a^2+b^2+c^2\le5\)

    Xảy ra khi \(a=2;b=1;c=0\) và hoán vị

      bởi Vũ Nam Khánh 14/01/2019
    Like (0) Báo cáo sai phạm

Nếu bạn hỏi, bạn chỉ thu về một câu trả lời.
Nhưng khi bạn suy nghĩ trả lời, bạn sẽ thu về gấp bội!

Lưu ý: Các trường hợp cố tình spam câu trả lời hoặc bị báo xấu trên 5 lần sẽ bị khóa tài khoản

Gửi câu trả lời Hủy
 
 

Các câu hỏi mới

NONE
OFF